2020-2021/TST/DM/2102_DM2/TST3/20_2102_DM2.tex

204 lines
7.5 KiB
TeX

\documentclass[a5paper,10pt]{article}
\usepackage{myXsim}
\usepackage{tasks}
% Title Page
\title{DM2 \hfill ZENAGUI Yanis}
\tribe{TST}
\date{\hfillÀ render pour le Mercredi 24 février}
\xsimsetup{
solution/print = false
}
\begin{document}
\maketitle
\begin{exercise}[subtitle={Loi binomiale}]
Trois personnes s'apprêtent à passer le portique de sécurité. On suppose que pour chaque personne la probabilité que le portique sonne est égale à $0.23$.
Soit $X$ la variable aléatoire donnant le nombre de personnes faisant sonner le portique, parmi les 3 personnes de ce groupe.
\begin{enumerate}
\item Tracer l'arbre représentant le situation.
\item Justifier que $X$ suit une loi binomiale dont on précisera les paramètres.
\item Quelle est la probabilité qu'une seule personne fasse sonner le portique?
\item Calculer puis interpréter les probabilités suivantes
\[
P(X = 0) \qquad \qquad P(X \geq 2)
\]
\item Calculer l'espérance de $X$ et interpréter le résultat.
\end{enumerate}
\end{exercise}
\begin{solution}
\begin{enumerate}
\item
\begin{tikzpicture}[sloped]
\node {.}
child {node {$0$}
child {node {$0$}
child {node {$0$}
edge from parent
node[above] {0.77}
}
child {node {$1$}
edge from parent
node[above] {0.23}
}
edge from parent
node[above] {0.77}
}
child[missing] {}
child {node {$1$}
child {node {$0$}
edge from parent
node[above] {0.77}
}
child {node {$1$}
edge from parent
node[above] {0.23}
}
edge from parent
node[above] {0.77}
}
edge from parent
node[above] {0.77}
}
child[missing] {}
child[missing] {}
child[missing] {}
child { node {$1$}
child {node {$0$}
child {node {$0$}
edge from parent
node[above] {0.77}
}
child {node {$1$}
edge from parent
node[above] {0.23}
}
edge from parent
node[above] {0.77}
}
child[missing] {}
child {node {$1$}
child {node {$0$}
edge from parent
node[above] {0.77}
}
child {node {$1$}
edge from parent
node[above] {0.23}
}
edge from parent
node[above] {0.77}
}
edge from parent
node[above] {0.23}
} ;
\end{tikzpicture}
\item Chaque personne a 2 possibilités (1: fait sonner ou 2: ne fait pas sonner) et l'on fait passer 3 personnes ce qui correspond à une répétition identique et aléatoire. On peut donc modéliser la situation par une loi binomiale.
\[
X \sim \mathcal{B}(3; 0.76)
\]
\item Probabilité qu'une seule personne fasse sonner le portique. On voit qu'il y a 3 branches qui correspondent à cette situation dont
\[
P(X = 1) = 3 \times 0.23^1 \times 0.77^2 \approx 0.409
\]
\item
\[
P(X = 0) = 0.77^3 \approx 0.457
\]
\[
P(X \geq 2) = P(X = 2) + P(X = 3) = 3 \times 0.23^2 \times 0.77^1 + 0.23^3 \approx 0.134
\]
\item Il faut d'abord tracer le tableau résumant la loi de probabilité:
\begin{center}
\begin{tabular}{|c|*{4}{c|}}
\hline
Valeur & 0 & 1 & 2 & 3 \\
\hline
Probabilité & $0.457$ & $0.409$ & $0.122$ &$0.012$ \\
\hline
\end{tabular}
\end{center}
On peut alors calculer l'espérance
\[
E[X] = 0 \times 0.457 + 1 \times 0.409 + 2 \times 0.122 + 3 \times 0.012 = 0.69
\]
On peut donc estimer qu'il y aura en moyenne $0.69$ personnes qui feront sonner le portique sur les 3 personnes.
\end{enumerate}
\end{solution}
\begin{exercise}[subtitle={Équation puissance}]
Résoudre les équations et inéquations suivantes
\begin{multicols}{2}
\begin{enumerate}
\item $10^x = 15$
\item $7^x = 38$
\item $0.66^x \leq 14$
\item $9 \times 0.36^x = 21$
\end{enumerate}
\end{multicols}
\end{exercise}
\begin{solution}
Les solutions ci-dessous ne sont pas justifiée car l'ordinateur ne sait pas faire. Par contre, vous vous devez savoir justifier vos réponses!
\begin{enumerate}
\item $x = \log(15)$
\item $x = \frac{\log(38)}{\log(7)}$
\item Il faut faire attention quand on divise par un log car ce dernier peut être négatif ce qui est le cas ici. Il faut donc pense à changer le sens de l'inégalité.
$x \geq \frac{\log(14)}{\log(0.66)}$
\item Il faut penser à faire la division à par $9$ avant d'utiliser le log car sinon, on ne peut pas utiliser la formule $\log(a^n) = n\times \log(a)$.
$x = \frac{\log(2.33)}{\log(0.36)}$
\end{enumerate}
\end{solution}
\begin{exercise}[subtitle={Étude de fonctions}]
Soit $f(x) = - 5x^3 + 375x^2 - 4515x + 9$ une fonction définie sur $\R$.
\begin{enumerate}
\item Calculer $f'(x)$ la dérivée de $f(x)$.
\item Calculer $f'(43)$ et $f'(7)$.
\item En déduire une forme factorisée de $f'(x)$.
\item Étudier le signe de $f'(x)$ et en déduire les variations de $f(x)$.
\item Est-ce que la fonction $f(x)$ admet un maximum ou un minimum? Si oui, calculer sa valeur.
\end{enumerate}
\end{exercise}
\begin{solution}
\begin{enumerate}
\item Dérivée de $f(x)$: $f'(x) = - 15x^2 + 750x - 4515$
\item
\begin{align*}
f'(43) &= - 15 \times 43^{2} + 750 \times 43 - 4515\\&= - 15 \times 1849 + 32250 - 4515\\&= - 27735 + 27735\\&= 0
\end{align*}
\begin{align*}
f'(7) &= - 15 \times 7^{2} + 750 \times 7 - 4515\\&= - 15 \times 49 + 5250 - 4515\\&= - 735 + 735\\&= 0
\end{align*}
Donc $x = 43$ et $x=7$ sont des racines de $f'(x) = - 15x^2 + 750x - 4515$.
\item On en déduit la forme factorisée suivante
\[
f'(x) = -15 (x - 43)(x-7)
\]
\item Pas de correction disponible
\item À causes des branches extérieurs, la fonction $f(x)$ n'a pas de maximum ou de minimum.
\end{enumerate}
\end{solution}
%\printsolutionstype{exercise}
\end{document}
%%% Local Variables:
%%% mode: latex
%%% TeX-master: "master"
%%% End: